8
$\begingroup$

Let $f:\mathbb{R}^2\to \mathbb{R}$ be $L$-Lipschitz. Let $f_\varepsilon:=f*\eta_\varepsilon$ be its smooth $\varepsilon$-mollification, where

  • $\eta_\varepsilon(x)=\frac{1}{C\varepsilon^2}\eta(|x|/\varepsilon)$ and
  • $\eta:\mathbb{R}\to\mathbb{R}$ is a standard smooth cut-off function taking values in $[0,1]$, integrating to $C$ and such that $\eta=1$ on $(-1,1)$, $\eta=0$ outside $(2,2)$, and $|\eta'|\leq 2$.

It is standard that $f_\varepsilon$ is also $L$-Lipschitz and that, as $\varepsilon\to 0$, $f_\varepsilon \to f$ pointwise and $Df_\varepsilon \to Df$ $\mathcal{L}^2$-a.e.

Clearly, $f$ restricted to $\mathbb{R}\times \{0\}$ is still Lipschitz, and hence (by Rademacher's theorem) for $\mathcal{H}^1$-a.e. $X\in P$ the derivative $\frac{\partial f}{\partial x}$ exists at $X$. I am trying to figure out if it is true that, for $f_\varepsilon$ defined as above (i.e. the mollification of $f$ on the whole $\mathbb{R}^2$), there exists a sequence $\varepsilon_j\searrow 0$ such that the following holds: $$ \frac{\partial f_{\varepsilon_j}}{\partial x}(X)\to \frac{\partial f}{\partial x}(X)\quad\quad\text{for $\mathcal{H}^{1}$-a.e. $X\in \mathbb{R}\times \{0\}$.} \label{1}\tag{$\star$} $$ Of course, if we parametrise the horizontal lines by their vertical component, i.e. $\mathbb{R}\times \{t\}$ for $t\in \mathbb{R}$, then Fubini implies that \eqref{1} holds for $\mathcal{L}^1$-a.e. $\mathbb{R}\times \{t\}$.

I am looking for either a proof that \eqref{1} holds in fact for all lines (with the sequence $(\varepsilon_j)$, and potentially also the mollifier $\eta$, depending on the line), or for an explicit counterexample.

Remark: I am aware of the fact that, for each given $t$, it is possibile (using Whitney's approximation theorem) to find a sequence of smooth functions (possibly different from the mollifications) such that the convergence holds on $\mathbb{R}\times\{t\}$, but here I am only interested in the sequence of the mollifications.

$\endgroup$
4
  • $\begingroup$ It seems to me that this question should be a bit clarified. I start with your fonction $f$. Now I call $g$ the restriction of $f$ to $\mathbb R\times\{0\}$, which is indeed also Lipschitz-continuous. Your question seems to be: is it true that $\partial_1 f_\epsilon(x,0)$ converges to $g'(x)$, a.e. in $\mathbb R$. In particular, you cannot restrict the $L^\infty$ function $\partial_1f$ to a set of measure $0$. A hunch would be that it is true when the mollifier is a tensor product $\rho_1(x_1)\rho_2(x_2)$ and more difficult or untrue in the other cases. $\endgroup$ Commented May 14 at 14:49
  • $\begingroup$ @Bazin The mollified function $f_\varepsilon$ is smooth, so its restriction to $\mathbb{R}\times 0$ makes sense everywhere, and I am asking if it converges to $(f|_{\mathbb{R}\times 0})'$, which also makes sense $\mathcal{H}^1$-a.e. as $f$ is Lipschitz. I suspect the answer to be negative, by the way. $\endgroup$ Commented May 16 at 21:29
  • $\begingroup$ I agree with the formulation in your comment, but not completely with the way your question is written. I would also support your suspicion, except maybe in the tensor product case where the mollifier is tailored on the Line of restriction. $\endgroup$ Commented May 18 at 16:45
  • $\begingroup$ Please stop editing your question hence and forth - every edit bumps it to the front page, which is a nuisance for other people. $\endgroup$ Commented Oct 16 at 6:03

2 Answers 2

0
$\begingroup$

What I want to write is too long for a comment, so let me write it as an answer. I want to point out a couple of basic facts related to your question. Let $f:\mathbb R^n\longrightarrow \mathbb R$ be a bounded measurable function such that its distribution-derivative $f'$ is also bounded measurable. I follow your notations (in $n$ dimensions) and set $$ f_\epsilon=f\ast\eta_\epsilon, \tag{1}$$ so that we have $ f_\epsilon(x) -f(x)=\int\bigl(f(x-\epsilon z)-f(x)\bigr) \eta(z)dz, $ yielding immediately $$ \sup_x\vert f_\epsilon(x) -f(x)\vert\le \epsilon L \int\vert z \eta(z)\vert dz. $$ The derivative estimate can be derived from the Lebesgue differentiation theorem: indeed we have $$ f'_\epsilon(x) -f'(x)= \int\bigl(f'(x-y)-f'(x)\bigr) \eta(y/\epsilon)dy\epsilon^{-n}, $$ so that, assuming that $\eta$ is supported in the unit ball, we get \begin{multline} \vert f'_\epsilon(x) -f'(x)\vert\le \epsilon^{-n}\Vert \eta\Vert_{L^\infty} \int_{\vert y-x\vert\le \epsilon}\vert f'(y)-f'(x)\vert dy \\\le \frac{C}{\vert B(x,\epsilon)\vert} \int_{B(x,\epsilon)}\vert f'(y)-f'(x)\vert dy. \end{multline} The Lebesgue differentiation theorem tells you that there is a set $\mathcal L$ (the Lebesgue points of $f'$) whose complement has zero Lebesgue measure such that for all $x\in \mathcal L$ $$ \lim_{\epsilon\rightarrow 0}\frac{1}{\vert B(x,\epsilon)\vert}\int_{B(x,\epsilon)}\vert f'(y)-f'(x)\vert dy=0. $$ Moreover you have obviously $ \Vert f'_\epsilon\Vert_{L^\infty}\le \Vert f'\Vert_{L^\infty}\Vert\eta\Vert_{L^1}. $

Note that the Lebesgue differentiation theorem is highly non-trivial and that the argument for the derivative works if you have only $f'$ locally integrable.

$\endgroup$
0
$\begingroup$

Here is a counterexample. Let $\eta,\eta_\varepsilon,f_\varepsilon$ be as in the question.

First we consider scales close to $1$; consider the unit squares $S_{n}=[10000n,10000n+1]\times[1,2]$, for integer $n$. Consider copies $\phi_n(x)=\phi(x-10000n)$ of some $1$-Lipschitz non-negative bump-function $\phi$ with integral $\lambda\in[0.001,1]$, so that $\phi_n$ is supported at $S_n$. Let $f_0=\sum_n\phi_{n}$ and $(f_0)_\varepsilon=f_0*\eta_\varepsilon$

The function $f_0$ is just $0$ in the $\mathbb{R}\times\{0\}$. But for any $\varepsilon\in[3,300]$, the derivative of $(f_0)_\varepsilon=f_0*\eta_\varepsilon$ is not too close to $0$, as $(f_0)_\varepsilon(x,0)$ has value $>10^{-10}$ when $x\in10000\mathbb{Z}$ and $(f_0)_\varepsilon(x,0)=0$ for $x\in10000\mathbb{Z}+5000$. This implies, taking into account that $(f_0)_\varepsilon$ is $1$-Lipschitz, that the set $\left\{x\in\mathbb{R};\left|\frac{\partial}{\partial x}(f_0)_\varepsilon(x,0)\right|>10^{-100}\right\}$ has measure $>10^{-100}$ in each interval of length $10000$.

Now let's go down to smaller scales. For each $N\in\{0,1,2,\dots\}$, let $S_{N,n}=\frac{S_N}{100^N}$ and $\phi_{N,n}(x)=100^{-N}\phi_{n}(100^Nx)$ (so, the graph of $\phi_{N,n}$ is the graph of $\phi_n$ but scaled $\frac{1}{100^N}$ with respect to the origin). Let $f=\sum_{N,n}\phi_{N,n}$. From now I write $f(x)$ instead of $f(x,0)$ to abbreviate, similarly with $f_\varepsilon$.

In order to conclude, it will be enough to check that, for each $\varepsilon\in(0,1)$, the set $\left\{x\in[0,10000];|f_\varepsilon'(x)|>10^{-100}\right\}$ has measure $>10^{100}$; that would imply that for no sequence $(\varepsilon_j)\to0$ can we have $f_{\varepsilon_j}'(x)\to f'(x)=0$ for a.e. $x\in \mathbb{R}$.

Indeed, fix $\varepsilon>0$ and choose $N$ such that $\varepsilon\in\left[\frac{3}{100^{N}},\frac{300}{100^{N}}\right]$. Note that for all $x\in\frac{10^{6}\mathbb{Z}}{100^N}$, we have $f_\varepsilon(x)=f_\varepsilon(0)\geq\frac{10^{-10}}{100^N}$: we have $f_\varepsilon(x)=f_\varepsilon(0)$ because the squares $S_{k,n}$ with $k\leq N-2$ are too far from the real line to affect $f_\varepsilon$, and for $k\geq N-1$ the squares $(S_{k,n})_{n\in\mathbb{Z}}$ repeat in patterns with period dividing $\frac{10^{6}}{100^N}$. And we have $f_\varepsilon(0)\geq\frac{10^{-10}}{100^N}$ as the integral of $\phi_{N,n}$ is in $\left[\frac{0.001\lambda}{10^{6N}},\frac{\lambda}{10^{6N}}\right]$ and the maximum of $\eta_\varepsilon$ is $10^{4N}$.

But for all $x\in \frac{10000\mathbb{Z}+5000}{100^N}$, we have $f_\varepsilon(x)\leq \frac{1}{2}f_\varepsilon(0)$, because the ball $B(x,2\varepsilon)$ cannot intersect $S_{k,n}$ for any $n$ if $k\leq N$), and the integrals of the functions $\phi_{k,n}$ with $k>N$ have much smaller mass than $\phi_{N,n}$.

Thus, in each interval of length $\frac{10000}{100^N}$, the variation of $f_\varepsilon$ is at least $\frac{10^{-11}}{100^N}$. As $f_\varepsilon$ is $1$-Lipschitz, this implies that $\{x\in\mathbb{R};|\frac{\partial}{\partial x}f_\varepsilon(x,0)|>10^{-100}\}$ has measure $>\frac{10^{-100}}{100^N}$ in each interval of length $\frac{10000}{100^N}$. So $\{x\in\mathbb{R};|\frac{\partial}{\partial x}f_\varepsilon(x,0)|>10^{-100}\}$ has measure $>10^{-100}$ in each interval of length $10000$, so we are done.

$\endgroup$

You must log in to answer this question.

Start asking to get answers

Find the answer to your question by asking.

Ask question

Explore related questions

See similar questions with these tags.